ALS algorithm

- Question one

Which of the following statements are true? A 45-year-old man presents to the Emergency Department (ED) with right arm and jaw pain which started an hour ago whilst he was playing squash. This is his 12-lead ECG. The ECG shows:
Select one or more option(s) and then Confirm.
A 12-lead ECG print out.
 
 
 
 

Feedback

That’s partly right. Have another go.

Feedback

That’s partly right.

There is a P wave before every QRS complex i.e. sinus rhythm. There is ST elevation across the chest leads (V2 – V6) as well as the lateral limb leads (aVL, I). These findings are consistent with an anterolateral ST elevation myocardial infarction (STEMI). There is no evidence of a bundle branch block (the QRS width is normal).

Feedback

That’s not right. Have another go.

Feedback

That’s not right.

There is a P wave before every QRS complex i.e. sinus rhythm. There is ST elevation across the chest leads (V2 – V6) as well as the lateral limb leads (aVL, I). These findings are consistent with an anterolateral ST elevation myocardial infarction (STEMI). There is no evidence of a bundle branch block (the QRS width is normal).

Feedback

That’s right.

There is a P wave before every QRS complex i.e. sinus rhythm. There is ST elevation across the chest leads (V2 – V6) as well as the lateral limb leads (aVL, I). These findings are consistent with an anterolateral ST elevation myocardial infarction (STEMI). There is no evidence of a bundle branch block (the QRS width is normal).

References

See chapter 4 of the ALS manual for further reading about the Acute Coronary Syndromes.

Settings

Font colour

default inverted high contrast high contrast inverted high contrast soft green on black

Sample text

text looks like thisTEXT LOOKS LIKE THIS

Accessible version

Glossary

Select the terms in the list to see a full definition.
Static glossary
Help Glossary Settings Menu Next 2 / 7 Back